Olá, Comunidade!

Vocês devem ter notado que o site ficou um período fora do ar (do dia 26 até o dia 30 de maio de 2024).

Consegui recuperar tudo, e ainda fiz um UPGRADE no servidor! Agora estamos em um servidor dedicado no BRASIL!
Isso vai fazer com que o acesso fique mais rápido (espero 🙏)

Já arrumei os principais bugs que aparecem em uma atualização!
Mas, se você encontrar alguma coisa diferente, que não funciona direito, me envie uma MP avisando que eu arranjo um tempo pra arrumar!

Vamos crescer essa comunidade juntos 🥰

Grande abraço a todos,
Prof. Caju

Maratonas de FísicaV Maratona de Física IME/ITA

Moderador: [ Moderadores TTB ]


LPavaNNN
2 - Nerd
Mensagens: 140
Registrado em: 03 Nov 2012, 23:14
Última visita: 01-08-20
Agradeceu: 3 vezes
Agradeceram: 84 vezes
Jan 2017 28 13:43

Re: V Maratona de Física IME/ITA

Mensagem não lida por LPavaNNN »

Solução Problema 20:
O movimento em questão é um MHS e portanto tem período:

[tex3]T=2\pi\cdot \sqrt{\frac{k}{m}}[/tex3]

é necessário então, descobrir k

[tex3]F_G=k\cdot x[/tex3]

[tex3]\frac{GMm}{R^2}=kR[/tex3]

[tex3]k=\frac{GMm}{R^3}[/tex3]

substituindo a massa do planeta por volume x densidade:

[tex3]k=Gm.\frac{4\pi R^3}{3R^3}[/tex3]

[tex3]k=\frac{4\pi Gm}{3}[/tex3]

substituindo na equação do período

[tex3]T=2\pi.\sqrt{\frac{3}{4\pi pG}}[/tex3]

no entanto, o enunciado pede o tempo para atravessar o planeta, ou seja, metade do período:

[tex3]t=\pi.\sqrt{\frac{3}{4\pi p G}}[/tex3]

[tex3]t=\sqrt{\frac{3\pi}{4pG}}[/tex3]

Resposta B

-------------------------------------------------------------------------------------------

Problema 21 (IME-2015/2016)
IME.jpg
IME.jpg (32.2 KiB) Exibido 8381 vezes
A figura acima mostra uma fonte luminosa e uma lente convergente, presas a molas idênticas, de massas desprezíveis e relaxadas. A fonte e a lente são colocadas em contato, provocando a mesma elongação nas três molas. Em seguida são soltas e movimentam-se sem atrito. Do instante inicial até o instante em que a fonte e a lente se encontram novamente, determine o tempo total em que a imagem formada é virtual.

Dados:

constante elastica das molas: [tex3]k = 20g/s^2[/tex3]

massa da fonte luminosa + suporte: [tex3]20g[/tex3]

massa da lente: [tex3]10g[/tex3]

elongação das molas no instante do contato: [tex3]10cm[/tex3]

distancia focal da lente: [tex3]26,25cm[/tex3]
Resposta

[tex3]\Delta t=2arccos(\frac{1}{4})+2arccos(\frac{3}{4})[/tex3]

Editado pela última vez por LPavaNNN em 28 Jan 2017, 13:43, em um total de 1 vez.
Lucas Pavan
Avatar do usuário

card0z0
iniciante
Mensagens: 8
Registrado em: 29 Jun 2017, 09:07
Última visita: 26-07-17
Agradeceram: 1 vez
Jun 2017 29 10:57

Re: V Maratona de Física IME/ITA

Mensagem não lida por card0z0 »

Muito bom essa campanha, Parabéns ao pessoal.
_____________________________________________________________________________
"O mundo da voltas, olhando aqui vejo que a circunferência do giro não confere com a minha :P "

Editado pela última vez por card0z0 em 29 Jun 2017, 10:57, em um total de 1 vez.
Avatar do usuário

Jvrextrue13
1 - Trainee
Mensagens: 77
Registrado em: 18 Jul 2020, 16:58
Última visita: 09-06-21
Jul 2020 29 16:30

Re: V Maratona de Física IME/ITA

Mensagem não lida por Jvrextrue13 »

Solução do Problema 21
Primeiro, vamos calcular as equações horárias de cada oscilante.
Fonte de luz:
[tex3]x_F=A.cos(w_F.t)\\A=10cm\rightarrow \text{Elongação da mola no instante inicial}\\w_F=\sqrt{\frac{K}{m}}=\sqrt{\frac{20}{20}}=1rad/s\\x_F=10.cos(t)[/tex3]

Lente Convergente:
[tex3]x_L=A.cos(w_L.t)\\A=10cm\\w_L=\sqrt{\frac{2K}{m}}=\sqrt{\frac{2.20}{10}}=2rad/s\\x_L=10.cos(2t)[/tex3]

Agora a parte de óptica da questão:
[tex3]\frac{1}{f}=\frac{1}{p}+\frac{1}{p'}\rightarrow p'=\frac{f.p}{p-f}>0\rightarrow Virtual\\p-f>0\\p < f[/tex3]
Bom, p representa a distância entra a lente e a fonte luminosa, e essa distância é dada por :
[tex3]p=20-x_F-x_L\\20-10cos(t)-10cos(2t)<26,2510^{-2}\\10(2-cos(t)-cos(2t))<26,25.10^{-2}\\2-cos(t)-cos^2(t)+sen^2(t)<26,25.10^{-1}\\-cos(t)-cos^2(t)+1-cos^2(t)<0,625\\-cos(t)-2cos^2(t)<-0,375\\y=cos(t)\\2y^2+y-\frac{3}{8}>0[/tex3]

Resolvendo essa inequação, encontramos

[tex3]cos(t)<-\frac{3}{4}\\ou\\cos(t)>\frac{1}{4}[/tex3]

Logo,

[tex3]\frac{1}{4}< cos(t)<1\\-1< cos(t)<-\frac{3}{4}[/tex3]
Aqui o que eu consegui tirar de conclusão foi que o sistema fonte-lente irá conjugar uma imagem virtual durante os intervalos de tempo achados e o total será a soma:
[tex3]T_{total}=2arccos\left(\frac{1}{4}\right)+2arccos\left(\frac{3}{4}\right)[/tex3]

Problema 22
IME(2018) A atmosfera densa de um planeta hipotético possui um índice de refração dependente das condições meteorológicas do local, tais como pressão, temperatura e umidade. Considere um modelo no qual a região da atmosfera é formada por [tex3]k+1[/tex3] camadas de índice de refração diferentes,[tex3]n_0, n_1, ... , n_k,[/tex3] de 1 km de altura cada, onde o índice de refração decai 10% a cada quilômetro de aumento na altitude.Considerando somente os efeitos da reflexão e da refração na atmosfera, se um raio luminoso, proveniente de um laser muito potente for disparado da superfície do planeta, formando um ângulo de [tex3]60º[/tex3] com a tangente à superfície, verifique se este raio alcançará o espaço, e, em caso negativo, determine qual será a altitude máxima alcançada pelo raio.
Dados:
-O planeta é esférico de raio [tex3]6370km[/tex3]
-[tex3]log(9)=0,95[/tex3]
-[tex3]log(2)=0,3[/tex3]
-[tex3]k=9[/tex3]
Anexos
21-0.jpg
21-0.jpg (34.66 KiB) Exibido 6580 vezes
Editado pela última vez por Jvrextrue13 em 30 Jul 2020, 14:18, em um total de 1 vez.
Ensinar/ajudar é uma das melhores formas de fixar o que já foi estudado :D
Avatar do usuário

Pedrolevi120
iniciante
Mensagens: 3
Registrado em: 30 Jul 2020, 12:50
Última visita: 04-07-21
Jul 2020 30 14:40

Re: V Maratona de Física IME/ITA

Mensagem não lida por Pedrolevi120 »

Resolução da questão 22

:arrow: Fatos importantes:

1.Lei dos senos
[tex3]\frac{a}{sen(A)}=\frac{b}{sen(B) }=\frac{c}{sen(C)}[/tex3]
2.Lei de Snell
[tex3]n_1.sen(\theta _1)=n_2.sen(\theta _2)[/tex3]

:arrow: O raio de luz sairá da superfície do planeta formando um ângulo de 60° com a normal, como mostra a figura:
Figura1.png
Figura1.png (21.93 KiB) Exibido 6557 vezes
Sabemos,pela lei de Snell,que:
[tex3]n_o.sen(\theta _o)=n_1.sen(\theta _1)[/tex3]
[tex3]n_0.sen(\theta _0)=0,9n_o.sen(\theta _1)[/tex3]
[tex3]sen(\theta _o)=0,9sen(\theta _1)[/tex3] [tex3]\rightarrow (1)[/tex3]

Sabemos,pela lei dos senos,que:
[tex3]\frac{R+1}{sen\alpha }=\frac{R}{sen\theta _O}\rightarrow (2)[/tex3]

Isolando [tex3]sen\theta _o[/tex3] em (2), substituindo em (1), e por fim isolando [tex3]sen\theta _1[/tex3] ,temos:
[tex3]sen(\theta _1)=\frac{Rsen\alpha }{0,9(R+1)}[/tex3]

Repetindo o mesmo processo para a refração da camada de indice [tex3]n_1[/tex3] para a camada de índice [tex3]n_2[/tex3] , e depois substituindo valor de [tex3]sen\theta _1 [/tex3] já obtido,chegaremos no seguinte resultado:

[tex3]sen\theta _2=\frac{sen\alpha .R}{(R+2).0,9^{2}}[/tex3]

Observando o padrão, podemos generalizar o seno do ângulo do raio refratado com a normal, para K camadas, da seguinte forma:
[tex3]sen(\theta _k)=sen\alpha .(\frac{R}{R+k}).\frac{1}{0,9^{k}}[/tex3]

Agora , É de fundamental importância notar que:
1.O ângulo [tex3]\alpha=30°\rightarrow sen\alpha =0,5[/tex3] .

2.Como k é no máximo 9,e R é 6370km,temos que R+k vale praticamente R.

Agora,como não sabemos em que camada o raio de luz refletirá totalmente, devemos considerar que o seno do ângulo [tex3]\theta _K[/tex3] vale 1,isto é, o raio sairá rasante com a camada em questão, portanto, temos que:
[tex3]1=\frac{1.6370}{2.6370.0,9^{k}}[/tex3]
[tex3]0,9^{k}=2^{-1}[/tex3]
Aplicando log em ambos os lados:
[tex3]k.log(0,9)=-1.log2[/tex3]
[tex3]k.(log9-log10)=-0,3[/tex3]
[tex3]k.(-0,05)=-0,3[/tex3]
[tex3]k=\frac{0,3}{0,05}[/tex3]
[tex3]k=6[/tex3]
Como cada camada tem 1km,e a reflexão total ocorre da 6° para a 7° camada,temos que a altura máxima atingida pelo raio até refletir totalmente mede 7km.



Problema 23

Um recipiente de vidro contendo gás tem uma lente convergente e uma fonte sonora presas a um
suporte (A) que desliza no trilho (B) a velocidade constante. Um feixe laser (C), que ilumina o objeto (D),
forma imagens reais nítidas por duas vezes em (E), separadas por uma diferença de tempo Δt, sendo
que, entre a formação dessas duas imagens, chegam n bips (pulsos sonoros de mesma duração) no
detector (F) e n − 1 bips são emitidos pela fonte sonora. Considerando que o comprimento do recipiente
é L e a distância focal da lente é f, determine a velocidade do som no gás.
questao.png
questao.png (164.63 KiB) Exibido 6557 vezes
Avatar do usuário

Tassandro
5 - Mestre
Mensagens: 1905
Registrado em: 15 Fev 2020, 17:01
Última visita: 03-10-23
Localização: Teresina, PI.
Agradeceu: 129 vezes
Agradeceram: 136 vezes
Jul 2020 30 15:34

Re: V Maratona de Física IME/ITA

Mensagem não lida por Tassandro »

Resolução do problema 23


Pela Equação de Gauss, podemos achar as posições das imagens:
[tex3]\frac 1f=\frac 1p+\frac 1{p'}\implies\frac 1f=\frac{1}{p}+\frac{1}{L-p}\implies\\
p^2-pL+fL=0\tag*{}[/tex3]
Resolvendo, vem que
[tex3]p_1=\frac{L+\sqrt{L^2-4fL}}{2},p_2=\frac{L-\sqrt{L^2-4fL}}{2}\tag*{}[/tex3]
Podemos observar que [tex3]p_1+p_2=L[/tex3] foi assumida como constante. Logo, a velocidade da fonte será dada
[tex3]v_{fonte}=\frac{p_1-p_2}{Δt}=\frac{\sqrt{L^2-4fL}}{Δt}[/tex3]
Pela Equação do Efeito Doppler,
[tex3]f_{obs}=f_{fonte}\(\frac{v_{som}\pm v_{obs}}{v_{som}\pm v_{fonte}}\)\implies\\
\frac{n}{Δt}=\frac{n-1}{Δt}\(\frac{v_{som}}{v_{som}-v_{fonte}}\)\implies \\
v_{som}=nv_{fonte}\therefore \boxed{v_{som}=\frac{n\sqrt{L^2-4fL}}{Δt}}[/tex3]


Problema 24

(ITA 2014) Partindo do repouso, uma bolinha cai verticalmente sobre um plano inclinado de um ângulo θ com relação à horizontal, originando seguidos choques perfeitamente elásticos. Se d é a distância inicial da bolinha ao plano, obtenha, em função de d, n e θ, a distância do ponto do n-ésimo choque em relação ao ponto do primeiro choque.
Editado pela última vez por Tassandro em 30 Jul 2020, 15:34, em um total de 1 vez.
Dias de luta, dias de glória.
Avatar do usuário

Jvrextrue13
1 - Trainee
Mensagens: 77
Registrado em: 18 Jul 2020, 16:58
Última visita: 09-06-21
Jul 2020 30 16:30

Re: V Maratona de Física IME/ITA

Mensagem não lida por Jvrextrue13 »

Solução do problema 24

Durante a primeira queda, de uma altura vertical h até o plano ela chegará com uma velocidade:
[tex3]V_o=\sqrt{2.g.h}\\h=\frac{d}{cos(\theta )}[/tex3]

Como as colisões sempre são perfeitamente elásticas, o ângulo de incidência do velocidade é o mesmo do vetor velocidade que "reflete" no plano inclinado,sendo assim os tempos entre as colisões sempre serão os mesmo. Vamos calcular o tempo entre duas colisões.
Decompondo a gravidade no eixos perpendicular e ao longo do plano inclinando tempos:

Na direção perpendicular ao plano inclinado:
[tex3]0=V_o.cos(\theta )-g.cos(\theta ).t_s\\t_{total}=2t_s=\frac{2V_o}{g}\\t_s\rightarrow \text{tempo de altura máxima em relação a superfície inclinada}[/tex3]

Agora, calculando a distância percorrida entre duas colisões n-ésimas:
[tex3]\Delta S=V_o.sen(\theta ).(t_{total}).(n-1)+gsen(\theta )\frac{[(t_{total})(n-1)]^2}{2}\\\Delta S=\frac{2V_o^2}{g}.sen(\theta )(n-1)+g.sen(\theta )\frac{4.V_o^2(n-1)^2}{2g^2}\\\Delta S=\frac{2.V_o^2.sen(\theta )(n-1)+sen(\theta ).2.V_o^2(n-1)^2}{g}\\\Delta S=\frac{2.V_o^2.sen(\theta )(n^2-n)}{g}\\\Delta S=\frac{2.2.g.h.sen(\theta )(n^2-n)}{g}=4hsen(\theta )(n^2-n)\\\Delta S=4d.tan(\theta )(n^2-n)[/tex3]


Problema 25

IME (2018) A figura acima mostra um dispositivo composto por um motor elétrico, cujo eixo se encontra ligado a uma polia ideal de raio [tex3]R[/tex3] , solidária a uma segunda polia de raio [tex3]r[/tex3] , sem deslizamento. Solidário ao segundo eixo há um disco rígido metálico de raio [tex3]r[/tex3] . Em duas extremidades opostas deste disco, foram fixados dois pêndulos compostos idênticos, com fios ideais e esferas homogêneas, de massa [tex3]m[/tex3] . Existe um fio extensível ligando as esferas inferiores, provendo uma força elástica que as mantém na configuração mostrada na figura. Determine, em função de [tex3]g, m, r[/tex3] e [tex3]R[/tex3] :
a) a velocidade angular [tex3]ω[/tex3] do motor elétrico;
b) a força elástica do fio extensível.
Dado:
• aceleração da gravidade: [tex3]g[/tex3]
Anexos
Anotação 2020-07-30 162830.png
Anotação 2020-07-30 162830.png (63.15 KiB) Exibido 6544 vezes
Editado pela última vez por Jvrextrue13 em 31 Jul 2020, 08:30, em um total de 1 vez.
Ensinar/ajudar é uma das melhores formas de fixar o que já foi estudado :D
Avatar do usuário

Pedrolevi120
iniciante
Mensagens: 3
Registrado em: 30 Jul 2020, 12:50
Última visita: 04-07-21
Jul 2020 31 11:07

Re: V Maratona de Física IME/ITA

Mensagem não lida por Pedrolevi120 »

Solução do problema 25

Como são dois fios, há dois valores de tração T1 e T2,segue a representação:
coiso.png
coiso.png (22.1 KiB) Exibido 6530 vezes
Escrevendo as equações da lei de newton para as duas bolinhas, atentando para os valores da resultante centrípeta:
1.Bolinha de baixo:
[tex3]T_1cos60+Fel=6rmw'^{2}\rightarrow Eq1[/tex3]
e
[tex3]T_1sen60=mg\rightarrow Eq2[/tex3]

2.Bolinha de cima:
[tex3]T_2cos60-T_1cos60=3rmw'^{2}\rightarrow Eq3[/tex3]
e
[tex3]T_2sen60=T_1sen60+mg\rightarrow Eq4[/tex3]

OBS:Os valores dos raios foram descobertos atraves do cosseno de 60 nos [tex3]\Delta AED [/tex3] e [tex3]\Delta CBA[/tex3] e depois somando o raio r da polia.

De Eq1,vem que:
[tex3]T_1=\frac{2mg\sqrt{3}}{3}[/tex3]

Substituindo o valor de [tex3]T_1[/tex3] em Eq4, e isolando [tex3]T_2[/tex3], temos:
[tex3]T_2=\frac{4mg\sqrt{3}}{3}[/tex3]

Substituindo o valor de [tex3]T_2 [/tex3] na Eq3, e isolando w', temos:
[tex3]w'=\frac{1}{3}\sqrt{\frac{g\sqrt{3}}{r}}[/tex3]

Da teoria de movimento circular, sabemos que as velocidades das duas polias de cima são iguais , logo:
[tex3]w'r=wR[/tex3]
[tex3]w=\frac{w'r}{R}[/tex3]
[tex3]w=\frac{r}{3R}\sqrt{\frac{g\sqrt{3}}{r}}[/tex3]

Isolando a Fel em Eq1, temos:
[tex3]Fel=\frac{\sqrt{3}}{3}mg[/tex3]


Problema 26
IME 2010/2011
Anotação 2020-07-31 110505.png
Anotação 2020-07-31 110505.png (157.14 KiB) Exibido 6530 vezes
miau rodo.png
miau rodo.png (40.47 KiB) Exibido 6530 vezes
Avatar do usuário

Tassandro
5 - Mestre
Mensagens: 1905
Registrado em: 15 Fev 2020, 17:01
Última visita: 03-10-23
Localização: Teresina, PI.
Agradeceu: 129 vezes
Agradeceram: 136 vezes
Jul 2020 31 12:54

Re: V Maratona de Física IME/ITA

Mensagem não lida por Tassandro »

Nota: O enunciado deve ser digitado

Resolução do Problema 26
20200731_121329.jpg
20200731_121329.jpg (21.33 KiB) Exibido 6519 vezes
a) É fácil ver que o módulo da velocidade relativa da carga em relação ao espelho vale 2v. Assim, [tex3]|v'|=2v[/tex3]
Completando os ângulos, temos que as velocidades relativas ao espelho valem
[tex3]v'_x=2v\sen(2α-90\degree)=-2vcos2α\\
v'_y=-2v\cos(2α-90\degree)=-2v\sen2α[/tex3]

Para um observador no solo,

[tex3]\boxed{v_x=-2v\cos2α+v\implies v_x=v(1-2\cos2α)\\
v_y=v'y=-2v\sen2α}[/tex3]

b) A aceleração centrípeta vale [tex3]a_{cp}=\frac{v^2}{L}[/tex3] na direção do eixo y, e a aceleração tangencial é causada pela componente horizontal da força elétrica, assim,
[tex3]F_{r_{t}}=ma_t\implies\frac{kQ^2\cosβ}{r^2}=ma_t\implies a_t=\frac{kQ^2\cosβ}{m\(\frac{d}{\senβ}\)^2}\therefore\boxed{a_t=\frac{kQ^2\sen^2β\cosβ}{md^2}}\\[/tex3]
20200731_121910.jpg
20200731_121910.jpg (29.01 KiB) Exibido 6519 vezes
c) Como o espelho está em MRU, a aceleração da imagem é a mesma no referencial do espelho e em um referencial em repouso.
Decompondo, temos que
[tex3]\begin{cases}a_x=a_{cp}\cos(2α-90\degree)+a_t\sen(2α-90\degree)\\
a_y=a_{cp}\sen(2α-90\degree)-a_t\cos(2α-90\degree)\end{cases}\\
\therefore a_x=\frac{v^2}{L}\sen2α-\frac{kQ^2\sen^2β\cosβ\cos2α}{md^2},
a_y=\frac{v^2}{L}\cos2α-\frac{kQ^2\sen^2β\cosβ\sen2α}{md^2}[/tex3]

Problema 27
20200731_125333.jpg
20200731_125333.jpg (6.73 KiB) Exibido 6519 vezes
(IME 2016) Uma corda de comprimento L e densidade linear constante gira em um plano em torno da
extremidade fixa no pontoA a umavelocidade angular constante igual a ω. Um pulso
ondulatório é gerado a partir de uma das extremidades. A velocidade vdo pulso, no referencial da corda, a uma distância r da extremidade fixa é dada por:
a)[tex3]ω\frac{(L-r)}{\sqrt2}[/tex3]
b)[tex3]ω\sqrt\frac{L(L-r)}{2}[/tex3]
c)[tex3]ω\frac{(L^2-r^2)}{\sqrt2L}[/tex3]
d)[tex3]ω\sqrt\frac{L^2-r^2}{2}[/tex3]
e)[tex3]ω\frac{L\sqrt{L-r}}{\sqrt{2(L+r)}}[/tex3]
Resposta

d
Anexos
20200731_121910.jpg
20200731_121910.jpg (29.01 KiB) Exibido 6519 vezes

Editado pela última vez por Tassandro em 31 Jul 2020, 12:56, em um total de 1 vez.
Dias de luta, dias de glória.
Responder
  • Tópicos Semelhantes
    Respostas
    Exibições
    Última mensagem

Voltar para “Maratonas de Física”